Exercícios Resolvidos de Força e Campo Gravitacional Envolvendo Integrais

Ver Teoria

Enunciado

A massa M está uniformemente distribuída ao longo de um disco de raio a . Determine o módulo, a direção e o sentido da força gravitacional entre o disco e a partícula de massa m localizada a uma distância x acima do centro do disco (Figura 12-39). O seu resultado se reduz a uma expressão correta quando x assume valores muito elevados? (Sugestão: Divida o disco em anéis finos concêntricos infinitesimais; a seguir, integre o resultado para achar a força total.)

Passo 1

Beleza, esse problema é um pouco grandinho, mas vamos por partes que vai dar tudo certo.

Primeiramente vamos encontrar o campo gravitacional gerado por um anel fino de raio R e massa m ' . Desenhando vamos ter isso aqui:

O que acontece é que pra cada pedacinho de massa do anel tem um outro pedacinho do outro lado e quando a gente soma a força gravitacional desses dois, a força resultante tem que apontar para baixo.

Isso significa que ao somarmos todas as contribuições de cada pedacinho do anel, a força total irá apontar para baixo também. Esse raciocínio é o que a gente chama de argumento de simetria!

Como a nossa força resultante aponta para baixo, na hora de somar todas as contribuições vamos levar em consideração somente a componente vertical. Assim, olhando a geometria do nosso problema, pra projetar cada força na direção vertical a gente tem que multiplicar d F → pelo cosseno de α . Pela nossa figura

c o s α = x r = x x 2 + R 2  

Passo 2

Outra coisa importante é que todos os pontos com massa d m ' desse anel estão à mesma distância r da massa m , assim a força exercida por cada pedacinho vai ser

d F = G m d m ' r 2

Mas como nos importa a componente vertical então temos que calcular

d F z = d F c o s α = G m d m ' r 2 x r

d F z = G m d m ' x x 2 + R 2 3

Aí agora temos que integrar esse cara pra calcular a força total

F z = ∫ G m d m ' x x 2 + R 2 3

Nós vamos integrar todos os pedacinhos do anel agora, ou seja, só d m ' . Como x , R , G e m são os mesmos pra todos os pedaços d m ' então são constantes na nossa integral, o que permite que a gente faça

F z = G m x x 2 + R 2 3 ∫ d m '

E o que é essa integral de d m ' ? É somente a massa total do nosso anel! Assim a força gravitacional exercida pelo anel será

F z = G m m ' x x 2 + R 2 3

Passo 3

Agora que já sabemos a força gravitacional para um anel genérico, temos que ajustar essa expressão encontrada para chegar na força exercida por um disco. Esse disco é composto por vários anéis e o raio desses anéis vai variar de 0 até a (raio do disco).

Pra isso então teremos que fazer uma nova integral que vai ser algo do tipo

F d i s c o = ∫ d F a n e l

Mas como ligar o anel com o disco? Bem, se eu disse que o que varia de um anel pro outro é o raio então temos que ter no final uma integral em R !

Pra isso tem o pulo do gato que é olhar pra massa de cada anel! A densidade de massa de todos os anéis é igual, concorda? Eles são feitos do mesmo material! O que muda de um pro outro é o raio, e consequentemente o seu comprimento. Assim anéis maiores vão ter mais massa! Vamos usar essa informação então

Agora eu vou supor que um dado anel vai ter um raio R e uma espessura beeeemm pequenina d R de forma que a sua área vai ser o seu comprimento vezes essa espessura

d A = 2 π R d R

Qual vai ser a massa desse anel? Ela vai ser agora um d M

d M = σ d A

Esse σ é a densidade superficial de massa do disco! Se sabemos que o disco tem massa M e área π a 2 , então essa densidade superficial de massa é

σ = M π a 2

Assim, a massa do anel será

d M = M π a 2 2 π R d R

d M = 2 M a 2 R d R

No fundo nós fizemos essa regra de 3

Passo 4

Finalmente estamos prontos pra montar a nossa última (eu prometo!) integral

F d i s c o = ∫ d F a n e l

Já calculamos a força de anel um anel de massa m '

F a n e l = G m m ' x x 2 + R 2 3

Como estamos falando de um anel de massa d M então vamos mudar para

d F a n e l = G m x x 2 + R 2 3 d M

Substituindo o d M do passo anterior

d F a n e l = G m x x 2 + R 2 3 2 M a 2 R d R

Jogando isso na nossa integral finalmente ficamos com

F d i s c o = ∫ G m x x 2 + R 2 3 2 M a 2 R d R

Finalmente temos uma integral com R !!!!!!

Agora, vamos lá! Os limites de integração são de 0 até a e como a integral é em R podemos tirar da integral tudo que é constante, assim

F d i s c o = 2 G M m x a 2 ∫ 0 a R d R x 2 + R 2 3

Passo 5

Agora é só fazer essa integralzinha aí rsrsrsrsrs

Você pode olhar no nosso material de cálculo onde tá tudo super explicadinho aqui! Eu vou fazer aqui, mas lá tem mais detalhes se você quiser dar uma relembrada, belezinha?

Essa integral é feita por substituição. Pra isso vamos definir uma nova variável u pra sumir com esse x 2 + R 2 do denominador, assim

u = x 2 + R 2

De forma que (lembrando que x é constante aqui!)

d u = 2 R d R → R d R = d u 2

E os limites de integração ficam

R = 0 → u = x 2

R = a → u = x 2 + a 2

Assim a nossa integral fica

∫ 0 a R d R x 2 + R 2 3 = ∫ x 2 x 2 + a 2 d u / 2 u 3 = 1 2 ∫ x 2 x 2 + a 2 u - 3 / 2 d u

1 2 - 2 u - 1 / 2 x 2 x 2 + a 2 = - 1 x 2 + a 2 - 1 x 2 = 1 x 2 - 1 x 2 + a 2

Passo 6

Pronto! Agora é só jogar na nossa expressão da força!

F d i s c o = 2 G M m x a 2 1 x 2 - 1 x 2 + a 2

Supondo x > 0 , vamos ter x 2 = x , assim

F d i s c o = 2 G M m x a 2 1 x - 1 x 2 + a 2

Botando o x fora do parênteses pra dentro

F d i s c o = 2 G M m a 2 1 - x x 2 + a 2

Show! Achamos a expressão pra força exercida pelo disco! Agora só falta botar a direção! Chamando - x ^ de direção para baixo teremos

F → d i s c o = 2 G M m a 2 1 - x x 2 + a 2 - x ^

Passo 7

Agora vamos ver se essa expressão faz sentido quando x assume valores muito elevados. O que significa isso? Isso significa tomar o limite de x indo pra infinito! Pra isso vamos olhar a parte que contém x na nossa expressão, ou seja, vamos olhar para o seguinte limite

lim x → ∞ ⁡ x x 2 + a 2

Pra fazer esse cara vamos reescrever a raiz como

x 2 + a 2 = x 2 1 + a 2 x 2 = x 1 + a 2 x 2

(Supondo x > 0 )

Assim o nosso limite será

lim x → ∞ ⁡ x x 2 + a 2 = lim x → ∞ ⁡ x x 1 + a 2 x 2 = lim x → ∞ ⁡ 1 1 + a 2 x 2 = 1 1 + 0 = 1

Então a nossa expressão pra força exercida pelo disco será

F → d i s c o x → ∞ = 2 G M m a 2 1 - 1 - x ^ = 0

Ou seja, láááááá no infinito a força exercida pelo disco vai ser nula, o que faz total sentido já que quando a gente tiver infinitamente afastado de um corpo a influência dele será nula! ;)

Resposta

F → d i s c o = 2 G M m a 2 1 - x x 2 + a 2 - x ^

Essa expressão vai para 0 no limite x → ∞ o que é o esperado quando dois corpos estão infinitamente afastados.

Exercícios de Livros Relacionados

Um fio homogêneo de massa M tem a forma de um anel circular de raio a . Calcule a força de atração gravitacional exercida pelo fio sobre uma partícula de massa m situada sobre o eixo (perpendicular ao
Um furo é feito da superfície da Terra até o seu centro, como na figura 11-30. Ignore a rotação da terra e a resistência do ar, e trate a Terra como uma esfera homogênea. (a) Qual é o trabalho necessá
Na Fig. 13-39, uma partícula de massa m 1 = 0,67 k g está a uma distância de 23 cm de uma das extremidades de uma barra homogênea de comprimento L = 3,0 m e massa M = 5,0 k g . Qual é o módulo da forç
Determine a energia potencial gravitacional da barra fina do Exemplo 11-8 e de uma partícula pontual de massa m 0 que está no eixo x em x = x 0 , onde x 0 ≥ 1 2 L . (a) Mostre que a energia potencial
Considere um fio retilíneo homogêneo de massa M e comprimento L e uma partícula de massa m alinhada com o fio, à distância D de uma extremidade. Mostre que a força de atração gravitacional exercida pe